Difference between revisions of "2011 AIME II Problems/Problem 15"

(Solution: Unnecessary information, and more strict vs. not strict)
(21 intermediate revisions by 10 users not shown)
Line 3: Line 3:
 
Let <math>P(x) = x^2 - 3x - 9</math>. A real number <math>x</math> is chosen at random from the interval <math>5 \le x \le 15</math>. The probability that <math>\lfloor\sqrt{P(x)}\rfloor = \sqrt{P(\lfloor x \rfloor)}</math> is equal to <math>\frac{\sqrt{a} + \sqrt{b} + \sqrt{c} - d}{e}</math> , where <math>a</math>, <math>b</math>, <math>c</math>, <math>d</math>, and <math>e</math> are positive integers. Find <math>a + b + c + d + e</math>.
 
Let <math>P(x) = x^2 - 3x - 9</math>. A real number <math>x</math> is chosen at random from the interval <math>5 \le x \le 15</math>. The probability that <math>\lfloor\sqrt{P(x)}\rfloor = \sqrt{P(\lfloor x \rfloor)}</math> is equal to <math>\frac{\sqrt{a} + \sqrt{b} + \sqrt{c} - d}{e}</math> , where <math>a</math>, <math>b</math>, <math>c</math>, <math>d</math>, and <math>e</math> are positive integers. Find <math>a + b + c + d + e</math>.
  
==Solution==
+
==Solution 1==
  
 
Table of values of <math>P(x)</math>:
 
Table of values of <math>P(x)</math>:
  
<math>\begin{array*}
+
<cmath>\begin{align*}
P(5) = 1 \\
+
P(5) &= 1 \\
P(6) = 9 \\
+
P(6) &= 9 \\
P(7) = 19 \\
+
P(7) &= 19 \\
P(8) = 31 \\
+
P(8) &= 31 \\
P(9) = 45 \\
+
P(9) &= 45 \\
P(10) = 61 \\
+
P(10) &= 61 \\
P(11) = 79 \\
+
P(11) &= 79 \\
P(12) = 99 \\
+
P(12) &= 99 \\
P(13) = 121 \\
+
P(13) &= 121 \\
P(14) = 145 \\
+
P(14) &= 145 \\
P(15) = 171 \\
+
P(15) &= 171 \\
\end{array*}</math>
+
\end{align*}</cmath>
  
In order for <math>\lfloor \sqrt{P(x)} \rfloor = \sqrt{P(\lfloor x \rfloor)}</math> to hold, <math>\sqrt{P(\lfloor x \rfloor)}</math> must be an integer and hence <math>P(\lfloor x \rfloor)</math> must be a perfect square. This limits <math>x</math> to <math>5 \le x < 6</math> or <math>6 \le x < 7</math> or <math>13 \le x < 14</math> since, from the table above, those are the only values of <math>x</math> for which <math>P(\lfloor x \rfloor)</math> is an perfect square. However, in order for <math>\sqrt{P(x)}</math> to be rounded down to <math>P(\lfloor x \rfloor)</math>, <math>P(x)</math> must be less than the next perfect square after <math>P(\lfloor x \rfloor)</math> (for the said intervals). Now, we consider the three difference cases.
+
In order for <math>\lfloor \sqrt{P(x)} \rfloor = \sqrt{P(\lfloor x \rfloor)}</math> to hold, <math>\sqrt{P(\lfloor x \rfloor)}</math> must be an integer and hence <math>P(\lfloor x \rfloor)</math> must be a perfect square. This limits <math>x</math> to <math>5 \le x < 6</math> or <math>6 \le x < 7</math> or <math>13 \le x < 14</math> since, from the table above, those are the only values of <math>x</math> for which <math>P(\lfloor x \rfloor)</math> is an perfect square. However, in order for <math>\sqrt{P(x)}</math> to be rounded down to <math>P(\lfloor x \rfloor)</math>, <math>P(x)</math> must be less than the next perfect square after <math>P(\lfloor x \rfloor)</math> (for the said intervals). Now, we consider the three cases:
  
  
 
Case <math>5 \le x < 6</math>:
 
Case <math>5 \le x < 6</math>:
  
<math>P(x)</math> must not be greater than the first perfect square after <math>1</math>, which is <math>4</math>. Since <math>P(x)</math> is increasing for <math>x \ge 5</math>, we just need to find where <math>P(x) = 4</math> and the values that will work will be <math>5 \le x < \text{root}</math>.
+
<math>P(x)</math> must be less than the first perfect square after <math>1</math>, which is <math>4</math>, ''i.e.'':
  
<math>\begin{array*}
+
<math>1 \le P(x) < 4</math> (because <math>\lfloor \sqrt{P(x)} \rfloor = 1</math> implies <math>1 \le \sqrt{P(x)} < 2</math>)
x^2 - 3x - 9 = 4 \\
+
 
x = \frac{3 + \sqrt{61}}{2}
+
Since <math>P(x)</math> is increasing for <math>x \ge 5</math>, we just need to find the value <math>v \ge 5</math> where <math>P(v) = 4</math>, which will give us the working range <math>5 \le x < v</math>.
\end{array*}</math>
+
 
 +
<cmath>\begin{align*}
 +
v^2 - 3v - 9 &= 4 \\
 +
v &= \frac{3 + \sqrt{61}}{2}
 +
\end{align*}</cmath>
  
 
So in this case, the only values that will work are <math>5 \le x < \frac{3 + \sqrt{61}}{2}</math>.
 
So in this case, the only values that will work are <math>5 \le x < \frac{3 + \sqrt{61}}{2}</math>.
Line 37: Line 41:
 
Case <math>6 \le x < 7</math>:
 
Case <math>6 \le x < 7</math>:
  
<math>P(x)</math> must not be greater than the first perfect square after <math>9</math>, which is <math>16</math>.
+
<math>P(x)</math> must be less than the first perfect square after <math>9</math>, which is <math>16</math>.
  
<math>\begin{array*}
+
<cmath>\begin{align*}
x^2 - 3x - 9 = 16 \\
+
v^2 - 3v - 9 &= 16 \\
x = \frac{3 + \sqrt{109}}{2}
+
v &= \frac{3 + \sqrt{109}}{2}
\end{array*}</math>
+
\end{align*}</cmath>
  
 
So in this case, the only values that will work are <math>6 \le x < \frac{3 + \sqrt{109}}{2}</math>.
 
So in this case, the only values that will work are <math>6 \le x < \frac{3 + \sqrt{109}}{2}</math>.
Line 48: Line 52:
 
Case <math>13 \le x < 14</math>:
 
Case <math>13 \le x < 14</math>:
  
<math>P(x)</math> must not be greater than the first perfect square after <math>121</math>, which is <math>144</math>.
+
<math>P(x)</math> must be less than the first perfect square after <math>121</math>, which is <math>144</math>.
  
<math>\begin{array*}
+
<cmath>\begin{align*}
x^2 - 3x - 9 = 144 \\
+
v^2 - 3v - 9 &= 144 \\
x = \frac{3 + \sqrt{621}}{2}
+
v &= \frac{3 + \sqrt{621}}{2}
\end{array*}</math>
+
\end{align*}</cmath>
  
 
So in this case, the only values that will work are <math>13 \le x < \frac{3 + \sqrt{621}}{2}</math>.
 
So in this case, the only values that will work are <math>13 \le x < \frac{3 + \sqrt{621}}{2}</math>.
Line 59: Line 63:
 
Now, we find the length of the working intervals and divide it by the length of the total interval, <math>15 - 5 = 10</math>:
 
Now, we find the length of the working intervals and divide it by the length of the total interval, <math>15 - 5 = 10</math>:
  
<math>\begin{array*}
+
<cmath>\begin{align*}
 
\frac{\left( \frac{3 + \sqrt{61}}{2} - 5 \right) + \left( \frac{3 + \sqrt{109}}{2} - 6 \right) + \left( \frac{3 + \sqrt{621}}{2} - 13 \right)}{10} \\
 
\frac{\left( \frac{3 + \sqrt{61}}{2} - 5 \right) + \left( \frac{3 + \sqrt{109}}{2} - 6 \right) + \left( \frac{3 + \sqrt{621}}{2} - 13 \right)}{10} \\
= \frac{\sqrt{61} + \sqrt{109} + \sqrt{621} - 39}{20}
+
&= \frac{\sqrt{61} + \sqrt{109} + \sqrt{621} - 39}{20}
\end{array*}</math>
+
\end{align*}</cmath>
 +
 
 +
Thus, the answer is <math>61 + 109 + 621 + 39 + 20 = \fbox{850}</math>.
 +
 
 +
P.S. You don't need to calculate all the values of P(x) calculated by the above solution. Some very simple modular arithmetic eliminates a large portion of the numbers. The time saved is not that much if you are already at your mathcounts prime.
 +
 
 +
== Solution 2 ==
 +
 
 +
Make the substitution <math>y=2x-3</math>, so <math>P(x)=\frac{y^2-45}{4}.</math> We're looking for solutions to <cmath>\bigg\lfloor{\sqrt{\frac{y^2-45}{4}}\bigg\rfloor}=\sqrt{\frac{\lfloor{y\rfloor}^2-45}{4}}</cmath>with the new bounds <math>y\in{[7,27]}</math>. Since the right side is an integer, it must be that <math>\frac{\lfloor{y\rfloor}^2-45}{4}</math> is a perfect square. For simplicity, write <math>\lfloor{y\rfloor}=a</math> and <cmath>a^2-45=4b^2\implies{(a-2b)(a+2b)=45}.</cmath>Since <math>a-2b<a+2b</math>, it must be that <math>(a-2b,a+2b)=(1,45),(3,15),(5,9)</math>, which gives solutions <math>(23,11),(9,3),(7,1)</math>, respectively. But this gives us three cases to check:
 +
 
 +
Case 1: <math>\bigg\lfloor{\sqrt{\frac{y^2-45}{4}}\bigg\rfloor}=11</math>.
 +
 
 +
In this case, we have <cmath>11\leq{\sqrt{\frac{y^2-45}{4}}}<12\implies{y\in{[23,\sqrt{621})}}.</cmath>
 +
Case 2: <math>\bigg\lfloor{\sqrt{\frac{y^2-45}{4}}\bigg\rfloor}=3</math>.
 +
 
 +
In this case, we have <cmath>3\leq{\sqrt{\frac{y^2-45}{4}}}<4\implies{y\in{[9,\sqrt{109})}}.</cmath>
 +
Case 3: <math>\bigg\lfloor{\sqrt{\frac{y^2-45}{4}}\bigg\rfloor}=1</math>
 +
 
 +
In this case, we have <cmath>1\leq{\sqrt{\frac{y^2-45}{4}}}<2\implies{y\in{[7,\sqrt{61})}}.</cmath>
 +
To finish, the total length of the interval from which we choose <math>y</math> is <math>27-7=20</math>. The total length of the success intervals is <cmath>(\sqrt{61}-7)+(\sqrt{109}-9)+(\sqrt{621}-23)=\sqrt{61}+\sqrt{109}+\sqrt{621}-39,</cmath>which means the probability is <cmath>\frac{\sqrt{61}+\sqrt{109}+\sqrt{621}-39}{20}.</cmath>AIMEifying the answer gives <math>\boxed{850}</math>.
 +
 
 +
== Solution 3 (Graphing) ==
 +
 
 +
It's definitely possible to conceptualize this problem visually, if that's your thing, since it is a geometric probability problem. Let <math>A = \lfloor\sqrt{P(x)}\rfloor</math> and <math>B = \sqrt{P(\lfloor x \rfloor)}</math>. The graph of <math>A</math> and <math>B</math> will look like this, with <math>A</math> having only integral y-values and <math>B</math> having only integral x-values:
 +
 
 +
[[File:2011 AIME II Problem 15 Graph 1.png|400px]]
 +
 
 +
As both <math>A</math> and <math>B</math> consist of a bunch of line segments, the probability that <math>A = B</math> is the "length" of the overlap between the segments of <math>A</math> and <math>B</math> divided by the total length of the segments of <math>B</math>.
 +
 
 +
Looking at the graph, we see that <math>A</math> and <math>B</math> will overlap only when <math>B</math> is an integer. Specifically, each region of overlap will begin when <math>\sqrt{P(x)}\ = k,  5 \le x \le 15</math> has solutions for integral <math>k</math> in the range of <math>A</math>, which consists of the integers <math>1-13</math>, and end when <math>A</math> jumps up to its next y-value.
 +
 
 +
Using the quadratic formula, we see that the start point of each of these overlapping segments will be the integral values of <math>\frac{3 + \sqrt{45 + 4k^2}}{2}</math> for <math>k</math> in the specified range, meaning <math>45 + 4k^2</math> must be a perfect square. Plugging in all the possible values of <math>k</math>, we get <math>k = 1, 3, 11</math>, corresponding to start points of <math>x = 5, 6, 13</math>. As already stated, the endpoints will occur when <math>A</math> jumps up to the next integer <math>k+1</math> at each of these segments, at which point the x-value will be <math>\frac{3 + \sqrt{45 + 4(k+1)^2}}{2}</math>. On the graph, the overlapping segments of <math>A</math> and <math>B</math> would be represented by the highlighted green segments below:
 +
 
 +
[[File:2011 AIME II Problem 15 Graph 2.png|400px]]
 +
 
 +
 
 +
Taking the difference between this second x-value and the start point for each of our start points <math>x = 5, 6, 13</math> and summing them will give us the total length of these green segments. We can then divide this value by ten (the total length of the segments of <math>B</math>) to give us the probability of overlap between <math>A</math> and <math>B</math>.
 +
 
 +
Doing so gives us:
 +
 
 +
<math>\frac{\left( \frac{3 + \sqrt{61}}{2} - 5 \right) + \left( \frac{3 + \sqrt{109}}{2} - 6 \right) + \left( \frac{3 + \sqrt{621}}{2} - 13 \right)}{10} = \frac{\sqrt{61} + \sqrt{109} + \sqrt{621} - 39}{20}</math>
 +
 
 +
<math>\implies{61 + 109 + 621 + 39 + 20 = \fbox{850}}</math>.
 +
 
 +
~ anellipticcurveoverq
 +
 
 +
==Solution 4==
 +
Note that all the "bounds" have to be less than the number+1, otherwise it wouldn't fit the answer format. Therefore, the answer is <math>\frac{3*3+\sqrt{9+4(4+9)}-10+\sqrt{9+4(16+9)}-12+\sqrt{9+4(144+9)}}{20} \implies \boxed{850}</math>
 +
 
 +
~Lcz
 +
 
 +
==See also==
 +
{{AIME box | year = 2011 | n = II | num-b=14 | after=Last Problem}}
  
So the answer is <math>61 + 109 + 621 + 39 + 20 = \fbox{850}</math>.
+
[[Category:Intermediate Combinatorics Problems]]
 +
{{MAA Notice}}

Revision as of 23:07, 16 June 2020

Problem

Let $P(x) = x^2 - 3x - 9$. A real number $x$ is chosen at random from the interval $5 \le x \le 15$. The probability that $\lfloor\sqrt{P(x)}\rfloor = \sqrt{P(\lfloor x \rfloor)}$ is equal to $\frac{\sqrt{a} + \sqrt{b} + \sqrt{c} - d}{e}$ , where $a$, $b$, $c$, $d$, and $e$ are positive integers. Find $a + b + c + d + e$.

Solution 1

Table of values of $P(x)$:

\begin{align*} P(5) &= 1 \\ P(6) &= 9 \\ P(7) &= 19 \\ P(8) &= 31 \\ P(9) &= 45 \\ P(10) &= 61 \\ P(11) &= 79 \\ P(12) &= 99 \\ P(13) &= 121 \\ P(14) &= 145 \\ P(15) &= 171 \\ \end{align*}

In order for $\lfloor \sqrt{P(x)} \rfloor = \sqrt{P(\lfloor x \rfloor)}$ to hold, $\sqrt{P(\lfloor x \rfloor)}$ must be an integer and hence $P(\lfloor x \rfloor)$ must be a perfect square. This limits $x$ to $5 \le x < 6$ or $6 \le x < 7$ or $13 \le x < 14$ since, from the table above, those are the only values of $x$ for which $P(\lfloor x \rfloor)$ is an perfect square. However, in order for $\sqrt{P(x)}$ to be rounded down to $P(\lfloor x \rfloor)$, $P(x)$ must be less than the next perfect square after $P(\lfloor x \rfloor)$ (for the said intervals). Now, we consider the three cases:


Case $5 \le x < 6$:

$P(x)$ must be less than the first perfect square after $1$, which is $4$, i.e.:

$1 \le P(x) < 4$ (because $\lfloor \sqrt{P(x)} \rfloor = 1$ implies $1 \le \sqrt{P(x)} < 2$)

Since $P(x)$ is increasing for $x \ge 5$, we just need to find the value $v \ge 5$ where $P(v) = 4$, which will give us the working range $5 \le x < v$.

\begin{align*} v^2 - 3v - 9 &= 4 \\ v &= \frac{3 + \sqrt{61}}{2} \end{align*}

So in this case, the only values that will work are $5 \le x < \frac{3 + \sqrt{61}}{2}$.

Case $6 \le x < 7$:

$P(x)$ must be less than the first perfect square after $9$, which is $16$.

\begin{align*} v^2 - 3v - 9 &= 16 \\ v &= \frac{3 + \sqrt{109}}{2} \end{align*}

So in this case, the only values that will work are $6 \le x < \frac{3 + \sqrt{109}}{2}$.

Case $13 \le x < 14$:

$P(x)$ must be less than the first perfect square after $121$, which is $144$.

\begin{align*} v^2 - 3v - 9 &= 144 \\ v &= \frac{3 + \sqrt{621}}{2} \end{align*}

So in this case, the only values that will work are $13 \le x < \frac{3 + \sqrt{621}}{2}$.

Now, we find the length of the working intervals and divide it by the length of the total interval, $15 - 5 = 10$:

\begin{align*} \frac{\left( \frac{3 + \sqrt{61}}{2} - 5 \right) + \left( \frac{3 + \sqrt{109}}{2} - 6 \right) + \left( \frac{3 + \sqrt{621}}{2} - 13 \right)}{10} \\ &= \frac{\sqrt{61} + \sqrt{109} + \sqrt{621} - 39}{20} \end{align*}

Thus, the answer is $61 + 109 + 621 + 39 + 20 = \fbox{850}$.

P.S. You don't need to calculate all the values of P(x) calculated by the above solution. Some very simple modular arithmetic eliminates a large portion of the numbers. The time saved is not that much if you are already at your mathcounts prime.

Solution 2

Make the substitution $y=2x-3$, so $P(x)=\frac{y^2-45}{4}.$ We're looking for solutions to \[\bigg\lfloor{\sqrt{\frac{y^2-45}{4}}\bigg\rfloor}=\sqrt{\frac{\lfloor{y\rfloor}^2-45}{4}}\]with the new bounds $y\in{[7,27]}$. Since the right side is an integer, it must be that $\frac{\lfloor{y\rfloor}^2-45}{4}$ is a perfect square. For simplicity, write $\lfloor{y\rfloor}=a$ and \[a^2-45=4b^2\implies{(a-2b)(a+2b)=45}.\]Since $a-2b<a+2b$, it must be that $(a-2b,a+2b)=(1,45),(3,15),(5,9)$, which gives solutions $(23,11),(9,3),(7,1)$, respectively. But this gives us three cases to check:

Case 1: $\bigg\lfloor{\sqrt{\frac{y^2-45}{4}}\bigg\rfloor}=11$.

In this case, we have \[11\leq{\sqrt{\frac{y^2-45}{4}}}<12\implies{y\in{[23,\sqrt{621})}}.\] Case 2: $\bigg\lfloor{\sqrt{\frac{y^2-45}{4}}\bigg\rfloor}=3$.

In this case, we have \[3\leq{\sqrt{\frac{y^2-45}{4}}}<4\implies{y\in{[9,\sqrt{109})}}.\] Case 3: $\bigg\lfloor{\sqrt{\frac{y^2-45}{4}}\bigg\rfloor}=1$

In this case, we have \[1\leq{\sqrt{\frac{y^2-45}{4}}}<2\implies{y\in{[7,\sqrt{61})}}.\] To finish, the total length of the interval from which we choose $y$ is $27-7=20$. The total length of the success intervals is \[(\sqrt{61}-7)+(\sqrt{109}-9)+(\sqrt{621}-23)=\sqrt{61}+\sqrt{109}+\sqrt{621}-39,\]which means the probability is \[\frac{\sqrt{61}+\sqrt{109}+\sqrt{621}-39}{20}.\]AIMEifying the answer gives $\boxed{850}$.

Solution 3 (Graphing)

It's definitely possible to conceptualize this problem visually, if that's your thing, since it is a geometric probability problem. Let $A = \lfloor\sqrt{P(x)}\rfloor$ and $B = \sqrt{P(\lfloor x \rfloor)}$. The graph of $A$ and $B$ will look like this, with $A$ having only integral y-values and $B$ having only integral x-values:

2011 AIME II Problem 15 Graph 1.png

As both $A$ and $B$ consist of a bunch of line segments, the probability that $A = B$ is the "length" of the overlap between the segments of $A$ and $B$ divided by the total length of the segments of $B$.

Looking at the graph, we see that $A$ and $B$ will overlap only when $B$ is an integer. Specifically, each region of overlap will begin when $\sqrt{P(x)}\ = k,  5 \le x \le 15$ has solutions for integral $k$ in the range of $A$, which consists of the integers $1-13$, and end when $A$ jumps up to its next y-value.

Using the quadratic formula, we see that the start point of each of these overlapping segments will be the integral values of $\frac{3 + \sqrt{45 + 4k^2}}{2}$ for $k$ in the specified range, meaning $45 + 4k^2$ must be a perfect square. Plugging in all the possible values of $k$, we get $k = 1, 3, 11$, corresponding to start points of $x = 5, 6, 13$. As already stated, the endpoints will occur when $A$ jumps up to the next integer $k+1$ at each of these segments, at which point the x-value will be $\frac{3 + \sqrt{45 + 4(k+1)^2}}{2}$. On the graph, the overlapping segments of $A$ and $B$ would be represented by the highlighted green segments below:

2011 AIME II Problem 15 Graph 2.png


Taking the difference between this second x-value and the start point for each of our start points $x = 5, 6, 13$ and summing them will give us the total length of these green segments. We can then divide this value by ten (the total length of the segments of $B$) to give us the probability of overlap between $A$ and $B$.

Doing so gives us:

$\frac{\left( \frac{3 + \sqrt{61}}{2} - 5 \right) + \left( \frac{3 + \sqrt{109}}{2} - 6 \right) + \left( \frac{3 + \sqrt{621}}{2} - 13 \right)}{10} = \frac{\sqrt{61} + \sqrt{109} + \sqrt{621} - 39}{20}$

$\implies{61 + 109 + 621 + 39 + 20 = \fbox{850}}$.

~ anellipticcurveoverq

Solution 4

Note that all the "bounds" have to be less than the number+1, otherwise it wouldn't fit the answer format. Therefore, the answer is $\frac{3*3+\sqrt{9+4(4+9)}-10+\sqrt{9+4(16+9)}-12+\sqrt{9+4(144+9)}}{20} \implies \boxed{850}$

~Lcz

See also

2011 AIME II (ProblemsAnswer KeyResources)
Preceded by
Problem 14
Followed by
Last Problem
1 2 3 4 5 6 7 8 9 10 11 12 13 14 15
All AIME Problems and Solutions

The problems on this page are copyrighted by the Mathematical Association of America's American Mathematics Competitions. AMC logo.png